Find existence of limit












3














Show that $left(frac{1}{|x|} +frac{1}{|y |}right)$ tends to infinity as $(x,y)$ tends to $(0,0)$.
I have used $x=rcos alpha$ and $y=rsin alpha$, $r>0$.But I got stuck as $left(frac{1}{|cos alpha|} +frac{1}{ |rsin alpha|}right) left(frac{1}{r}right)$ lies between $frac{1}{r}$ and infinity.










share|cite|improve this question




















  • 3




    "I got stuck as" has a mistake with the $r$ in the denominator.
    – Teepeemm
    Dec 13 at 20:31
















3














Show that $left(frac{1}{|x|} +frac{1}{|y |}right)$ tends to infinity as $(x,y)$ tends to $(0,0)$.
I have used $x=rcos alpha$ and $y=rsin alpha$, $r>0$.But I got stuck as $left(frac{1}{|cos alpha|} +frac{1}{ |rsin alpha|}right) left(frac{1}{r}right)$ lies between $frac{1}{r}$ and infinity.










share|cite|improve this question




















  • 3




    "I got stuck as" has a mistake with the $r$ in the denominator.
    – Teepeemm
    Dec 13 at 20:31














3












3








3







Show that $left(frac{1}{|x|} +frac{1}{|y |}right)$ tends to infinity as $(x,y)$ tends to $(0,0)$.
I have used $x=rcos alpha$ and $y=rsin alpha$, $r>0$.But I got stuck as $left(frac{1}{|cos alpha|} +frac{1}{ |rsin alpha|}right) left(frac{1}{r}right)$ lies between $frac{1}{r}$ and infinity.










share|cite|improve this question















Show that $left(frac{1}{|x|} +frac{1}{|y |}right)$ tends to infinity as $(x,y)$ tends to $(0,0)$.
I have used $x=rcos alpha$ and $y=rsin alpha$, $r>0$.But I got stuck as $left(frac{1}{|cos alpha|} +frac{1}{ |rsin alpha|}right) left(frac{1}{r}right)$ lies between $frac{1}{r}$ and infinity.







calculus limits






share|cite|improve this question















share|cite|improve this question













share|cite|improve this question




share|cite|improve this question








edited Dec 16 at 11:54









Jam

4,94711431




4,94711431










asked Dec 13 at 14:32









Kashmira

463




463








  • 3




    "I got stuck as" has a mistake with the $r$ in the denominator.
    – Teepeemm
    Dec 13 at 20:31














  • 3




    "I got stuck as" has a mistake with the $r$ in the denominator.
    – Teepeemm
    Dec 13 at 20:31








3




3




"I got stuck as" has a mistake with the $r$ in the denominator.
– Teepeemm
Dec 13 at 20:31




"I got stuck as" has a mistake with the $r$ in the denominator.
– Teepeemm
Dec 13 at 20:31










4 Answers
4






active

oldest

votes


















8














Note that



$$frac1{|x|}+frac1{|y|}ge frac1{|x|}to infty$$






share|cite|improve this answer





























    4














    Take the individual limits of each component. x and y are tending towards zero (the direction doesn't matter due to the magnitudes) so $frac{1}{|x|}$ and $frac{1}{|y|}$ both tend to infinity, and as a result so does their sum.



    EDIT:
    As suggested in the comments, this is an expansion as to why you can seperate the limits.



    $$lim_{(x,y)to(0,0)}{bigg(frac{1}{|x|}+frac{1}{|y|}bigg)}=lim_{(x,y)to(0,0)}{bigg(frac{1}{|x|}bigg)}+lim_{(x,y)to(0,0)}{bigg(frac{1}{|y|}bigg)}$$



    Examining the LHS we see that the first limit is independent of y, and the second independent of x. As a result we can rewrite the equation as follows:



    $$=lim_{xto0}{bigg(frac{1}{|x|}bigg)}+lim_{yto0}{bigg(frac{1}{|y|}bigg)}$$



    As stated above, the magnitudes mean that both limits tend to infinity as x and y tend to zero from either direction, so their sum is necessarily also infinity.






    share|cite|improve this answer



















    • 2




      You are right, but a few more words about why taking "individual limits of each component" can replace $(x,y)to (0,0)$ would be helpful in the present circumstance.
      – hardmath
      Dec 13 at 14:44










    • Thank you for the advice, I'll add an edit expanding on that.
      – M.M.
      Dec 14 at 10:06



















    2














    Hint:



    Use the fact that if $|(x,y)| < epsilon$, then $|x|<epsilon$ and $|y|<epsilon$






    share|cite|improve this answer





























      1














      We have:



      $(x,y) rightarrow 0.$



      Let $epsilon_n =1/n$, $n$ positive integer, be given.



      Then $|(x^2+y^2)^{1/2}| < delta_n$ $(=epsilon_n)$ implies



      $|(x^2+y^2)^{1/2}| < epsilon_n =1/n$, or



      $n=1/epsilon_n lt dfrac{1}{(x^2+y^2)^{1/2}}.$



      Note:



      $(x^2+y^2)^{1/2} ge |x|$, and
      $(x^2+y^2)^{1/2} ge |y|.$



      Then



      $dfrac{2}{(x^2+y^2)^{1/2}} le dfrac{1}{|x|}+dfrac{1}{|y|}.$



      And finally :



      $|(x^2+y^2)^{1/2}| lt delta_n$ implies



      $2n =2/epsilon_n lt dfrac{2}{(x^2+y^2)^{1/2}} le $



      $dfrac{1}{|x|} +dfrac{1}{|y|}.$






      share|cite|improve this answer























        Your Answer





        StackExchange.ifUsing("editor", function () {
        return StackExchange.using("mathjaxEditing", function () {
        StackExchange.MarkdownEditor.creationCallbacks.add(function (editor, postfix) {
        StackExchange.mathjaxEditing.prepareWmdForMathJax(editor, postfix, [["$", "$"], ["\\(","\\)"]]);
        });
        });
        }, "mathjax-editing");

        StackExchange.ready(function() {
        var channelOptions = {
        tags: "".split(" "),
        id: "69"
        };
        initTagRenderer("".split(" "), "".split(" "), channelOptions);

        StackExchange.using("externalEditor", function() {
        // Have to fire editor after snippets, if snippets enabled
        if (StackExchange.settings.snippets.snippetsEnabled) {
        StackExchange.using("snippets", function() {
        createEditor();
        });
        }
        else {
        createEditor();
        }
        });

        function createEditor() {
        StackExchange.prepareEditor({
        heartbeatType: 'answer',
        autoActivateHeartbeat: false,
        convertImagesToLinks: true,
        noModals: true,
        showLowRepImageUploadWarning: true,
        reputationToPostImages: 10,
        bindNavPrevention: true,
        postfix: "",
        imageUploader: {
        brandingHtml: "Powered by u003ca class="icon-imgur-white" href="https://imgur.com/"u003eu003c/au003e",
        contentPolicyHtml: "User contributions licensed under u003ca href="https://creativecommons.org/licenses/by-sa/3.0/"u003ecc by-sa 3.0 with attribution requiredu003c/au003e u003ca href="https://stackoverflow.com/legal/content-policy"u003e(content policy)u003c/au003e",
        allowUrls: true
        },
        noCode: true, onDemand: true,
        discardSelector: ".discard-answer"
        ,immediatelyShowMarkdownHelp:true
        });


        }
        });














        draft saved

        draft discarded


















        StackExchange.ready(
        function () {
        StackExchange.openid.initPostLogin('.new-post-login', 'https%3a%2f%2fmath.stackexchange.com%2fquestions%2f3038078%2ffind-existence-of-limit%23new-answer', 'question_page');
        }
        );

        Post as a guest















        Required, but never shown

























        4 Answers
        4






        active

        oldest

        votes








        4 Answers
        4






        active

        oldest

        votes









        active

        oldest

        votes






        active

        oldest

        votes









        8














        Note that



        $$frac1{|x|}+frac1{|y|}ge frac1{|x|}to infty$$






        share|cite|improve this answer


























          8














          Note that



          $$frac1{|x|}+frac1{|y|}ge frac1{|x|}to infty$$






          share|cite|improve this answer
























            8












            8








            8






            Note that



            $$frac1{|x|}+frac1{|y|}ge frac1{|x|}to infty$$






            share|cite|improve this answer












            Note that



            $$frac1{|x|}+frac1{|y|}ge frac1{|x|}to infty$$







            share|cite|improve this answer












            share|cite|improve this answer



            share|cite|improve this answer










            answered Dec 13 at 14:37









            gimusi

            1




            1























                4














                Take the individual limits of each component. x and y are tending towards zero (the direction doesn't matter due to the magnitudes) so $frac{1}{|x|}$ and $frac{1}{|y|}$ both tend to infinity, and as a result so does their sum.



                EDIT:
                As suggested in the comments, this is an expansion as to why you can seperate the limits.



                $$lim_{(x,y)to(0,0)}{bigg(frac{1}{|x|}+frac{1}{|y|}bigg)}=lim_{(x,y)to(0,0)}{bigg(frac{1}{|x|}bigg)}+lim_{(x,y)to(0,0)}{bigg(frac{1}{|y|}bigg)}$$



                Examining the LHS we see that the first limit is independent of y, and the second independent of x. As a result we can rewrite the equation as follows:



                $$=lim_{xto0}{bigg(frac{1}{|x|}bigg)}+lim_{yto0}{bigg(frac{1}{|y|}bigg)}$$



                As stated above, the magnitudes mean that both limits tend to infinity as x and y tend to zero from either direction, so their sum is necessarily also infinity.






                share|cite|improve this answer



















                • 2




                  You are right, but a few more words about why taking "individual limits of each component" can replace $(x,y)to (0,0)$ would be helpful in the present circumstance.
                  – hardmath
                  Dec 13 at 14:44










                • Thank you for the advice, I'll add an edit expanding on that.
                  – M.M.
                  Dec 14 at 10:06
















                4














                Take the individual limits of each component. x and y are tending towards zero (the direction doesn't matter due to the magnitudes) so $frac{1}{|x|}$ and $frac{1}{|y|}$ both tend to infinity, and as a result so does their sum.



                EDIT:
                As suggested in the comments, this is an expansion as to why you can seperate the limits.



                $$lim_{(x,y)to(0,0)}{bigg(frac{1}{|x|}+frac{1}{|y|}bigg)}=lim_{(x,y)to(0,0)}{bigg(frac{1}{|x|}bigg)}+lim_{(x,y)to(0,0)}{bigg(frac{1}{|y|}bigg)}$$



                Examining the LHS we see that the first limit is independent of y, and the second independent of x. As a result we can rewrite the equation as follows:



                $$=lim_{xto0}{bigg(frac{1}{|x|}bigg)}+lim_{yto0}{bigg(frac{1}{|y|}bigg)}$$



                As stated above, the magnitudes mean that both limits tend to infinity as x and y tend to zero from either direction, so their sum is necessarily also infinity.






                share|cite|improve this answer



















                • 2




                  You are right, but a few more words about why taking "individual limits of each component" can replace $(x,y)to (0,0)$ would be helpful in the present circumstance.
                  – hardmath
                  Dec 13 at 14:44










                • Thank you for the advice, I'll add an edit expanding on that.
                  – M.M.
                  Dec 14 at 10:06














                4












                4








                4






                Take the individual limits of each component. x and y are tending towards zero (the direction doesn't matter due to the magnitudes) so $frac{1}{|x|}$ and $frac{1}{|y|}$ both tend to infinity, and as a result so does their sum.



                EDIT:
                As suggested in the comments, this is an expansion as to why you can seperate the limits.



                $$lim_{(x,y)to(0,0)}{bigg(frac{1}{|x|}+frac{1}{|y|}bigg)}=lim_{(x,y)to(0,0)}{bigg(frac{1}{|x|}bigg)}+lim_{(x,y)to(0,0)}{bigg(frac{1}{|y|}bigg)}$$



                Examining the LHS we see that the first limit is independent of y, and the second independent of x. As a result we can rewrite the equation as follows:



                $$=lim_{xto0}{bigg(frac{1}{|x|}bigg)}+lim_{yto0}{bigg(frac{1}{|y|}bigg)}$$



                As stated above, the magnitudes mean that both limits tend to infinity as x and y tend to zero from either direction, so their sum is necessarily also infinity.






                share|cite|improve this answer














                Take the individual limits of each component. x and y are tending towards zero (the direction doesn't matter due to the magnitudes) so $frac{1}{|x|}$ and $frac{1}{|y|}$ both tend to infinity, and as a result so does their sum.



                EDIT:
                As suggested in the comments, this is an expansion as to why you can seperate the limits.



                $$lim_{(x,y)to(0,0)}{bigg(frac{1}{|x|}+frac{1}{|y|}bigg)}=lim_{(x,y)to(0,0)}{bigg(frac{1}{|x|}bigg)}+lim_{(x,y)to(0,0)}{bigg(frac{1}{|y|}bigg)}$$



                Examining the LHS we see that the first limit is independent of y, and the second independent of x. As a result we can rewrite the equation as follows:



                $$=lim_{xto0}{bigg(frac{1}{|x|}bigg)}+lim_{yto0}{bigg(frac{1}{|y|}bigg)}$$



                As stated above, the magnitudes mean that both limits tend to infinity as x and y tend to zero from either direction, so their sum is necessarily also infinity.







                share|cite|improve this answer














                share|cite|improve this answer



                share|cite|improve this answer








                edited Dec 14 at 10:12

























                answered Dec 13 at 14:35









                M.M.

                637




                637








                • 2




                  You are right, but a few more words about why taking "individual limits of each component" can replace $(x,y)to (0,0)$ would be helpful in the present circumstance.
                  – hardmath
                  Dec 13 at 14:44










                • Thank you for the advice, I'll add an edit expanding on that.
                  – M.M.
                  Dec 14 at 10:06














                • 2




                  You are right, but a few more words about why taking "individual limits of each component" can replace $(x,y)to (0,0)$ would be helpful in the present circumstance.
                  – hardmath
                  Dec 13 at 14:44










                • Thank you for the advice, I'll add an edit expanding on that.
                  – M.M.
                  Dec 14 at 10:06








                2




                2




                You are right, but a few more words about why taking "individual limits of each component" can replace $(x,y)to (0,0)$ would be helpful in the present circumstance.
                – hardmath
                Dec 13 at 14:44




                You are right, but a few more words about why taking "individual limits of each component" can replace $(x,y)to (0,0)$ would be helpful in the present circumstance.
                – hardmath
                Dec 13 at 14:44












                Thank you for the advice, I'll add an edit expanding on that.
                – M.M.
                Dec 14 at 10:06




                Thank you for the advice, I'll add an edit expanding on that.
                – M.M.
                Dec 14 at 10:06











                2














                Hint:



                Use the fact that if $|(x,y)| < epsilon$, then $|x|<epsilon$ and $|y|<epsilon$






                share|cite|improve this answer


























                  2














                  Hint:



                  Use the fact that if $|(x,y)| < epsilon$, then $|x|<epsilon$ and $|y|<epsilon$






                  share|cite|improve this answer
























                    2












                    2








                    2






                    Hint:



                    Use the fact that if $|(x,y)| < epsilon$, then $|x|<epsilon$ and $|y|<epsilon$






                    share|cite|improve this answer












                    Hint:



                    Use the fact that if $|(x,y)| < epsilon$, then $|x|<epsilon$ and $|y|<epsilon$







                    share|cite|improve this answer












                    share|cite|improve this answer



                    share|cite|improve this answer










                    answered Dec 13 at 14:34









                    5xum

                    89.5k393161




                    89.5k393161























                        1














                        We have:



                        $(x,y) rightarrow 0.$



                        Let $epsilon_n =1/n$, $n$ positive integer, be given.



                        Then $|(x^2+y^2)^{1/2}| < delta_n$ $(=epsilon_n)$ implies



                        $|(x^2+y^2)^{1/2}| < epsilon_n =1/n$, or



                        $n=1/epsilon_n lt dfrac{1}{(x^2+y^2)^{1/2}}.$



                        Note:



                        $(x^2+y^2)^{1/2} ge |x|$, and
                        $(x^2+y^2)^{1/2} ge |y|.$



                        Then



                        $dfrac{2}{(x^2+y^2)^{1/2}} le dfrac{1}{|x|}+dfrac{1}{|y|}.$



                        And finally :



                        $|(x^2+y^2)^{1/2}| lt delta_n$ implies



                        $2n =2/epsilon_n lt dfrac{2}{(x^2+y^2)^{1/2}} le $



                        $dfrac{1}{|x|} +dfrac{1}{|y|}.$






                        share|cite|improve this answer




























                          1














                          We have:



                          $(x,y) rightarrow 0.$



                          Let $epsilon_n =1/n$, $n$ positive integer, be given.



                          Then $|(x^2+y^2)^{1/2}| < delta_n$ $(=epsilon_n)$ implies



                          $|(x^2+y^2)^{1/2}| < epsilon_n =1/n$, or



                          $n=1/epsilon_n lt dfrac{1}{(x^2+y^2)^{1/2}}.$



                          Note:



                          $(x^2+y^2)^{1/2} ge |x|$, and
                          $(x^2+y^2)^{1/2} ge |y|.$



                          Then



                          $dfrac{2}{(x^2+y^2)^{1/2}} le dfrac{1}{|x|}+dfrac{1}{|y|}.$



                          And finally :



                          $|(x^2+y^2)^{1/2}| lt delta_n$ implies



                          $2n =2/epsilon_n lt dfrac{2}{(x^2+y^2)^{1/2}} le $



                          $dfrac{1}{|x|} +dfrac{1}{|y|}.$






                          share|cite|improve this answer


























                            1












                            1








                            1






                            We have:



                            $(x,y) rightarrow 0.$



                            Let $epsilon_n =1/n$, $n$ positive integer, be given.



                            Then $|(x^2+y^2)^{1/2}| < delta_n$ $(=epsilon_n)$ implies



                            $|(x^2+y^2)^{1/2}| < epsilon_n =1/n$, or



                            $n=1/epsilon_n lt dfrac{1}{(x^2+y^2)^{1/2}}.$



                            Note:



                            $(x^2+y^2)^{1/2} ge |x|$, and
                            $(x^2+y^2)^{1/2} ge |y|.$



                            Then



                            $dfrac{2}{(x^2+y^2)^{1/2}} le dfrac{1}{|x|}+dfrac{1}{|y|}.$



                            And finally :



                            $|(x^2+y^2)^{1/2}| lt delta_n$ implies



                            $2n =2/epsilon_n lt dfrac{2}{(x^2+y^2)^{1/2}} le $



                            $dfrac{1}{|x|} +dfrac{1}{|y|}.$






                            share|cite|improve this answer














                            We have:



                            $(x,y) rightarrow 0.$



                            Let $epsilon_n =1/n$, $n$ positive integer, be given.



                            Then $|(x^2+y^2)^{1/2}| < delta_n$ $(=epsilon_n)$ implies



                            $|(x^2+y^2)^{1/2}| < epsilon_n =1/n$, or



                            $n=1/epsilon_n lt dfrac{1}{(x^2+y^2)^{1/2}}.$



                            Note:



                            $(x^2+y^2)^{1/2} ge |x|$, and
                            $(x^2+y^2)^{1/2} ge |y|.$



                            Then



                            $dfrac{2}{(x^2+y^2)^{1/2}} le dfrac{1}{|x|}+dfrac{1}{|y|}.$



                            And finally :



                            $|(x^2+y^2)^{1/2}| lt delta_n$ implies



                            $2n =2/epsilon_n lt dfrac{2}{(x^2+y^2)^{1/2}} le $



                            $dfrac{1}{|x|} +dfrac{1}{|y|}.$







                            share|cite|improve this answer














                            share|cite|improve this answer



                            share|cite|improve this answer








                            edited Dec 13 at 17:05

























                            answered Dec 13 at 16:57









                            Peter Szilas

                            10.6k2720




                            10.6k2720






























                                draft saved

                                draft discarded




















































                                Thanks for contributing an answer to Mathematics Stack Exchange!


                                • Please be sure to answer the question. Provide details and share your research!

                                But avoid



                                • Asking for help, clarification, or responding to other answers.

                                • Making statements based on opinion; back them up with references or personal experience.


                                Use MathJax to format equations. MathJax reference.


                                To learn more, see our tips on writing great answers.





                                Some of your past answers have not been well-received, and you're in danger of being blocked from answering.


                                Please pay close attention to the following guidance:


                                • Please be sure to answer the question. Provide details and share your research!

                                But avoid



                                • Asking for help, clarification, or responding to other answers.

                                • Making statements based on opinion; back them up with references or personal experience.


                                To learn more, see our tips on writing great answers.




                                draft saved


                                draft discarded














                                StackExchange.ready(
                                function () {
                                StackExchange.openid.initPostLogin('.new-post-login', 'https%3a%2f%2fmath.stackexchange.com%2fquestions%2f3038078%2ffind-existence-of-limit%23new-answer', 'question_page');
                                }
                                );

                                Post as a guest















                                Required, but never shown





















































                                Required, but never shown














                                Required, but never shown












                                Required, but never shown







                                Required, but never shown

































                                Required, but never shown














                                Required, but never shown












                                Required, but never shown







                                Required, but never shown







                                Popular posts from this blog

                                Список кардиналов, возведённых папой римским Каликстом III

                                Deduzione

                                Mysql.sock missing - “Can't connect to local MySQL server through socket”